There are 30 students in Mr. Hall class. 1/3 of the students are traveling somewhere this summer. Of those traveling, 1/5 are going out of the country. How many students are traveling out of the country?

Answers

Answer 1

The number of students traveling are 2 students out of the 30 who are traveling out of the country this summer.  If 1/3 of the students are traveling somewhere this summer, then we can calculate the number of students who are traveling by multiplying the total number of students by 1/3:

Number of students traveling = 30 x 1/3 = 10  

Now, we need to find out how many of these 10 students are traveling out of the country. We know that 1/5 of the students who are traveling are going out of the country, so we can find the number of students who are traveling out of the country by multiplying the total number of traveling students by 1/5:

Number of students traveling out of the country = 10 x 1/5 = 2

Therefore, there are 2 students out of the 30 who are traveling out of the country this summer.

It's important to note that fractions can be converted to decimals or percentages to make them easier to work with. For example, 1/3 can be written as 0.33 or 33%, and 1/5 can be written as 0.20 or 20%. This can be particularly useful when dealing with more complex problems or when working with larger numbers.

In summary, by using the information given, we can determine that out of the 30 students in Mr. Hall's class, 10 are traveling somewhere this summer, and out of those 10 students, 2 are going out of the country. This type of problem-solving helps build math skills that are applicable in real-world scenarios, such as budgeting and planning for travel.

To know more about  traveling click here:

brainly.com/question/17235551

#SPJ4


Related Questions

Construct triange ABC, in which AB = 6 cm, angle BAC = 96 degrees and angle ABC = 35 degrees. Measure the length of BC. Give your answer to 1 d. P

Answers

From the construction of the triangle ABC we get that the measure length of BC is approximately 4.22cm

To construct triangle ABC, we can follow these steps:

Draw a line segment AB of length 6 cm.Draw an angle of 96 degrees at point A using a protractor.Draw an angle of 35 degrees at point B using a protractor.The intersection point of the two lines that were drawn in step 2 and 3 will be point C, which is the third vertex of the triangle.

To measure the length of BC in triangle ABC, we can use the law of sines.

The law of sines states that in any triangle ABC:

a / sin(A) = b / sin(B) = c / sin(C)

Where a, b, and c are the lengths of the sides of the triangle opposite to the angles A, B, and C, respectively.

In our triangle ABC, we know AB = 6 cm, angle BAC = 96 degrees and angle ABC = 35 degrees. We can find the measure of angle ACB by using the fact that the sum of the angles in a triangle is 180 degrees:

angle ACB = 180 - angle BAC - angle ABC

= 180 - 96 - 35 = 49 degrees

Now, we can apply the law of sines to find the length of BC:

BC / sin(35) = 6 / sin(96)

BC = 6 × sin(35) / sin(96)

Using a calculator, we can evaluate this expression to get:

BC ≈ 4.22 cm

Therefore, the length of BC in triangle ABC is approximately 4.22 cm.

To learn more about triangle click here

brainly.com/question/2773823

#SPJ4

if the measure of and acute angle is represented by x, then the measure of the angle that it is complementary which is represented by 90-x

Answers

The measure of the angle that it is complementary which is represented by 90-x is always true. Option A

What is an acute angle?

An acute angle is simply defined as an angle that measures from 90° and 0°. This means that it is smaller than a right angle.

It is formed in the space between two intersecting lines or planes, or from the intersection of two shapes.

What is a complementary angle?

A complementary angle can be defined as a pair of angles whose sum is equal or equivalent to 90 degrees.

From the information given, we have that;

x is the acute angle

The complementary angle is 90 - x

We can see that the angle x must be complementary to be subtracted from 90 degrees.

Learn about acute angles at: https://brainly.com/question/28958927

#SPJ1

The complete question:

If the measure of an acute angle is represented by x, then the measure of its complement is represented by 90 – X.

always true

sometimes true

never true



Question

The average of three numbers is 16. If one of the numbers is 18, what is the sum of the other two

numbers?

12

14

20

30

Answers

If the average of three numbers is 16 and one of the numbers is 18, then the sum of the other two numbers is option (d) 30

Let's use algebra to solve this problem. Let x and y be the other two numbers we are looking for. We know that the average of the three numbers is 16, so we can write:

(18 + x + y) / 3 = 16

Multiplying both sides by 3, we get,

[(18 + x + y) / 3] × 3 = 16 ×3

18 + x + y = 48

Subtracting 18 from both sides, we get,

18 + x + y - 18 = 48

x + y = 30

Therefore, the correct option is (d) 30

Learn more about average here

brainly.com/question/1416374

#SPJ4

Sally has 3:4 as many beads as Kelly. Kelly has 18 more beads than Sally. Find the average number of beads the girl have

Answers

The average number of beads that the girls have is 63

Let's start by using algebra to represent the given information:

Let b be the number of beads that Sally has.

Then, Kelly has 3/4 times as many beads as Sally, which can be expressed as (3/4)b.

Also, we know that Kelly has 18 more beads than Sally, which can be expressed as (b + 18).

Putting these together, we can write the equation:

(3/4)b = b + 18

Solving for b, we get:

b = 72

So, Sally has 72 beads, and Kelly has (3/4) × 72 = 54 beads.

The average number of beads that the girls have is (72 + 54)/2 = 63 beads

Learn more about average here

brainly.com/question/16956746

#SPJ4

Baseball hats are on sale for 12% off the original price of the sale price is $12.50 what was the original price? round the answer to the nearest cent

Answers

The original price of the baseball hat before 12% off in the sale was $14.20.

What is a percentage?

A number can be expressed as a fraction of 100 using a percentage. It frequently serves to indicate a portion of a total and is represented by the sign %. We may state that 25% of the class is made up of guys, for instance, if there are 100 pupils in the class and 25 of them are male.

The Roman term per centum, which meaning "by the hundred," is where the word "percent" originates.

Let us suppose the original price of the baseball hat = x.

Given that, baseball hats are on sale for 12% off the original price.

That is,

12.50 = x(1 - 12/100)

12.50 = x(100 - 12)/100

12.50 = x(0.88)

x = 14.2

Hence, the original price of the baseball hat was $14.20.

Learn more about percent here:

https://brainly.com/question/29763752

#SPJ1

PTC is a substance that has a strong bitter taste for some people and is tasteless for others. The ability to taste PTC is inherited and depends on a single gene that codes for a taste receptor on the tongue. Interestingly, although the PTC molecule is not found in nature, the ability to taste it correlates strongly with the ability to taste other naturally occurring bitter substances, many of which are toxins. About 75 % of Italians can taste PTC. You want to estimate the proportion of Americans with at least one Italian grandparent who can taste PTC. (a) Starting with the 75 % estimate for Italians, how large a sample must you collect in order to estimate the proportion of PTC tasters within ± 0.1 with 90 % confidence? (Enter your answer as a whole number.) n = (b) Estimate the sample size required if you made no assumptions about the value of the proportion who could taste PTC. (Enter your answer as a whole number.) n =

Answers

(a) Starting with the 75% estimate for Italians, the sample you must collect in order to estimate the proportion of PTC tasters within ± 0.1 with 90 % confidence is n = 51.

(b) The sample size required if you made no assumptions about the value of the proportion who could taste PTC is n = 68.

(a) To estimate the sample size needed to find the proportion of PTC tasters within ± 0.1 with 90% confidence, we will use the formula for sample size estimation in proportion problems:
n = (Z² * p * (1-p)) / E²

Where n is the sample size, Z is the Z-score corresponding to the desired confidence level (1.645 for 90% confidence), p is the proportion of PTC tasters (0.75), and E is the margin of error (0.1).

n = (1.645² * 0.75 * (1-0.75)) / 0.1²
n = (2.706 * 0.75 * 0.25) / 0.01
n ≈ 50.74


Since we need a whole number, we round up to the nearest whole number:
n = 51

(b) If no assumptions were made about the proportion of PTC tasters, we would use the worst-case scenario, which is p = 0.5 (maximum variance):

n = (1.645² * 0.5 * (1-0.5)) / 0.1²
n = (2.706 * 0.5 * 0.5) / 0.01
n ≈ 67.65

Again, rounding up to the nearest whole number:
n = 68

Learn more about margin of error here: https://brainly.com/question/29328438

#SPJ11

Let Y be a binomial random variable with n trials and probability of success given by p. Use the method of moment-generating functions to show that U = n - Y is a binomial random variable with n trials and probability of success given by 1 - p.

Answers

U is a binomial random variable with n trials and probability of success given by 1 - p.

As Y is a binomial random variable with n trials and probability of success given by p. Using the moment-generating functions method, it can be shown that U = n - Y is a binomial random variable with n trials and probability of success given by 1 - p. The binomial distribution is described by two parameters: n, which is the number of trials, and p, which is the probability of success in any given trial. If a binomial random variable is denoted by Y, then:[tex]P(Y = k) = \binom{n}{k}p^{k}(1 - p)^{n-k}[/tex]

The method of generating moments can be used to show that U = n - Y is a binomial random variable with n trials and probability of success given by 1 - p. The moment-generating function of a binomial random variable is given by: [tex]M_{y}(t) = [1 - p + pe^{t}]^{n}[/tex]

The moment-generating function for U is: [tex]M_{u}(t) = E(e^{tu}) = E(e^{t(n-y)})[/tex]

Using the definition of moment-generating functions, we can write: [tex]M_{u}(t) = E(e^{t(n-y)})$$$$= \sum_{y=0}^{n} e^{t(n-y)} \binom{n}{y} p^{y} (1-p)^{n-y}[/tex]

Taking the summation of the above expression: [tex]= \sum_{y=0}^{n} e^{tn} e^{-ty} \binom{n}{y} p^{y} (1-p)^{n-y}$$$$= e^{tn} \sum_{y=0}^{n} \binom{n}{y} (pe^{-t})^{y} [(1-p)^{n-y}]^{1}$$$$= e^{tn} (pe^{-t} + 1 - p)^{n}[/tex]

Comparing this expression with the moment-generating function for a binomial random variable, we can say that U is a binomial random variable with n trials and probability of success given by 1 - p.

To learn more about "Binomial random variable": brainly.com/question/30657837

#SPJ11

7) Roy buys pizza for his friends. A whole pizza costs P 190. 00 and P 40. 00 for every
additional topping. If he spent P 1070 for pizza with 3 sets of additional toppings, how
many whole pizzas did he buy?

Answers

Roy purchased whole pizzas for P 190.00 each. To determine the number of whole pizzas he bought, we can divide the total cost of the pizzas by the cost of each pizza. Therefore, the calculation P 950.00 / P 190.00 results in 5, indicating that Roy bought five whole pizzas.

Roy spent a total of P 1070 for pizza with 3 sets of additional toppings. Since each set of additional toppings costs P 40.00, then the total cost of the toppings is 3 x P 40.00 = P 120.00. Subtracting this from the total amount spent gives us P 950.00, which is the cost of the pizzas alone.

Since each whole pizza costs P 190.00, we can divide the cost of the pizzas by the cost of each pizza to find the number of whole pizzas Roy bought. Therefore, P 950.00 / P 190.00 = 5.

Thus, Roy bought 5 whole pizzas.

Learn more about arithmetic here: brainly.com/question/11559160

#SPJ4

1 Write a positive or negative number to represent each change in the high temperature. Tuesday's high temperature was 4 degrees less than Monday's high temperature. ​

Answers

Thursday’s high temperature was 6.5 degrees more than Wednesday’s high temperature.

Average temperature is calculated as (Monday's temperature plus Tuesday's temperature plus Wednesday's temperature plus Thursday's temperature)/Total days.

Temporary for Monday and Tuesday plus Temperatures for Wednesday and Thursday.

The sum of the temperatures for Monday, Tuesday, Wednesday, and Thursday is equal.

As stated, the 6.5 degree average for the days of Tuesday, Wednesday, Thursday, and Friday.

using the formula no.

Average temperature is equal to (Tuesday's temperature plus Wednesday's temperature plus Thursday's temperature plus Friday's temperature)/Total days.

Thus, Thursday’s high temperature was 6.5 degrees more than Wednesday’s high temperature.

To know more about temperature, click the below link

https://brainly.com/question/11464844

#SPJ4

the complete question is

Write a positive or negative number to represent each change in the high temperature.

1. Tuesday’s high temperature was 4 degrees less than Monday’s high temperature. ?

2. Wednesday’s high temperature was 3.5 degrees less than Tuesday’s high temperature. ?

3. Thursday’s high temperature was 6.5 degrees more than Wednesday’s high temperature. ?

4. Friday’s high temperature was 2 degrees less than Thursday’s high temperature. ?

The accurate scale diagram shows a telephone mast and a box.
Find an estimate for the real height, in metres, of the telephone mast.
telephone mast
5.5
+2.5 m
box
+
Total marks: 2

Answers

Using proportions, the real height of the telephone mast is estimated to be 9 meters.

What exactly is a proportion?

A proportion is a fraction of a total amount, and equations are constructed using these fractions and estimates to find the desired measures in the problem using basic arithmetic operations like multiplication and division. Because the telephone box and the mast are similar figures in this problem, their side lengths are proportional.

The following proportional relationship is established as a result:

x / 1.5 cm = 10.8 cm / 1.8 cm.

The relationship's left side can be simplified as follows:

6 = x / 1.5 cm.

The estimate is then calculated using cross multiplication, as shown below:

6 x 1.5 cm = 9.5 cm².

To know more about fraction, visit:

https://brainly.com/question/10354322

#SPJ1

a family of five recently replaced its 5-gallon-per-minute showerheads with water-saving 2-gallon-per-minute showerheads. each member of the family averages 8 minutes in the shower per day. in a 30-day period, how many fewer gallons of water will the family use with the new showerheads? responses 60 60 800 800 2,400 2,400 3,600 3,600 7,200

Answers

The family will use 3,600 fewer gallons of water with the new showerheads in a 30-day period.

A family of five recently replaced its 5-gallon-per-minute showerheads with water-saving 2-gallon-per-minute showerheads. Each member of the family averages 8 minutes in the shower per day. In a 30-day period, the family will use 3,600 fewer gallons of water with the new showerheads. How many fewer gallons of water will the family use with the new showerheads?

Calculate the number of gallons used per person per shower before:

5 gal/min x 8 min = 40 gallons per person per shower.

The number of gallons used per person per shower after:

2 gal/min x 8 min = 16 gallons per person per shower.

The number of gallons used per person per day before:

40 gallons x 1 shower = 40 gallons per person per day.

The number of gallons used per person per day after:

16 gallons x 1 shower = 16 gallons per person per day

The number of gallons used per family per day before:

40 gallons x 5 people = 200 gallons per day

The number of gallons used per family per day after:

16 gallons x 5 people = 80 gallons per day

The number of gallons used per month before:

40 gallons x 5 people x 30 days = 6,000 gallons per month

The number of gallons used per month after:

16 gallons x 5 people x 30 days = 2,400 gallons per month

The family will use 6,000 - 2,400 = 3,600 fewer gallons of water with the new showerheads in a 30-day period. Therefore, the answer is 3,600.

To learn more about water gallon problems refer :

https://brainly.com/question/14700882

#SPJ11

Maria purchased 1,000 shares of stock for $35. 50 per share in 2014. She sold them in 2016 for $55. 10 per share. Express her capital gain as a percent, rounded to the nearest tenth of a percent

Answers

Maria's capital gain is 55.21%. Rounded to the nearest tenth of a percent, this is 55.2%.

To determine Maria's capital gain as a percent, we need to calculate the difference between the selling price and the purchase price, and then express this difference as a percentage of the purchase price.

The purchase price for 1,000 shares of stock was:

$35.50 x 1,000 = $35,500

The selling price for 1,000 shares of stock was:

$55.10 x 1,000 = $55,100

The capital gain is the difference between the selling price and the purchase price:

$55,100 - $35,500 = $19,600

To express this gain as a percentage of the purchase price, we divide the capital gain by the purchase price and multiply by 100:

($19,600 / $35,500) x 100 = 55.21%

In summary, to calculate the percent capital gain from the purchase and selling price of a stock, we simply divide the difference between the two prices by the purchase price and multiply by 100.

To learn more about capital gain click on,

https://brainly.com/question/28628208

#SPJ4

The length of a rectangle is increasing at a rate of 7 cm/s and its width is increasing at a rate of 8 cm/s. When the length is 7 cm and the width is 5 cm, how fast is the area of the rectangle increasing (in cm²/s)?

Answers

Answer:

Step-by-step explanation: In the problem, they tell us that

dL / dt = 7 cm/s (the rate at which the length is changing) and

dw / dt = 8 cm/s (the rate at which the width is changing)

Want dA/dt (the rate at which the area is changing) when L = 7 cm and w = 5 cm

The equation for the area of a rectangle is:

A = L·w, so will need the product rule when taking the derivative.

dA/dt = L (dw/dt) + w (dL/dt)

Now just plug in all of the given numbers:

dA/dt = (7)(7) + (5)(8) = 49+40 = 89  cm²/s

Can you help me with this?

Answers

16. The equatiοn οf the line in slοpe-intercept fοrm that passes thrοugh the pοint (-6, 5) and is parallel tο x + 2y = 14 is y = (-1/2)x + 2.

What is equatiοn οf line?

The equatiοn οf a straight line is y = mx + c, y = m x + c m is the gradient and c is the height at which the line crοsses the y -axis, alsο knοwn as the y -intercept.

16. Tο write the equatiοn οf a line in slοpe-intercept fοrm, we need tο find the slοpe and the y-intercept οf the line.

Tο find the slοpe οf the line, we can rewrite the equatiοn x + 2y = 14 in slοpe-intercept fοrm y = mx + b by sοlving fοr y:

x + 2y = 14

2y = -x + 14

y = (-1/2)x + 7

The slοpe οf the line is -1/2.

Since the line we want tο find is parallel tο this line, it will have the same slοpe οf -1/2.

Nοw we can use the pοint-slοpe fοrm οf the equatiοn οf a line tο find the equatiοn οf the line that passes thrοugh the pοint (-6, 5) with a slοpe οf -1/2:

y - y1 = m(x - x1)

where (x1, y1) is the pοint (-6, 5), and m is the slοpe, -1/2.

y - 5 = (-1/2)(x - (-6))

y - 5 = (-1/2)x - 3

y = (-1/2)x + 2

17. The equation perpendicular to y = -(2/3)x + 4, passing through (-4, 6)

perpendicular equations slope would be negative reciprocal to the current line.

The slope in y = -(2/3)x + 4, is m = -(2/3),

The negative reciprocal of -(2/3) is 3/2

Now, applying the x and y values in pοint-slοpe fοrm

y - 6 = 3/2(x - (-4))

y =  3/2(x+4) + 6

y =  (3/2)x + 6 + 6

y =  (3/2)x + 12

18. Since the line we want tο find is parallel tο this line, it will have the same slοpe.

Lets find the slope using slope formula

[tex]\rm m = \dfrac{y_2 - y_1}{x_2 - x_1}[/tex]

[tex]\rm m = \dfrac{0 - (-1)}{2 - (-1)}[/tex]

[tex]\rm m = \dfrac{1}{3}[/tex]

Now, using the point slope form

y - 1 = 1/3(x - 3)

y = 1/3(x - 3) + 1

y = (1/3)x - 1 + 1

y = (1/3)x

Learn more about equation of line on:

https://brainly.com/question/18831322

#SPJ1

the ratio of students who ade the honor roll to the total number of stoudents is 1:50. if there are 500 students in total how many made the honor roll?

Answers

If there are 500 students in total, the number of students who made the honor roll is 10 students, given that the ratio of students who made the honor roll to the total number of students is 1:50.

The number of students who made the honor roll can be found using proportions. Here's how to do it:

Let X be the number of students who made the honor roll.

The proportion can be set up using the given ratio as follows:

1:50 = X:500

Cross-multiplying this equation and solving for X gives:

50X = 500

X = 10

Therefore, 10 students made the honor roll.

Learn more about proportions here: https://brainly.com/question/18437927

#SPJ11

For each problem, select the best response (a) A x2 statistic provides strong evidence in favor of the alternative hypothesis if its value is A. a large positive number. OB. exactly 1.96 c. a large negative number. D. close to o E. close to 1. (b) A study was performed to examine the personal goals of children in elementary school. A random sample of students was selected and the sample was given a questionnaire regarding achieving personal goals. They were asked what they would most like to do at school: make good grades, be good at sports, or be popular. Each student's sex (boy or girl) was also recorded. If a contingency table for the data is evaluated with a chi-squared test, what are the hypotheses being tested? A. The null hypothesis that boys are more likely than girls to desire good grades vs. the alternative that girls are more likely than boys to desire good grades. OB. The null hypothesis that sex and personal goals are not related vs. the alternative hypothesis that sex and personal goals are related. C. The null hypothesis that there is no relationship between personal goals and sex vs. the alternative hypothesis that there is a positive, linear relationship. OD. The null hypothesis that the mean personal goal is the same for boys and girls vs. the alternative hypothesis is that the means differ. O E. None of the above. (C) The variables considered in a chi-squared test used to evaluate a contingency table A. are normally distributed. B. are categorical. C. can be averaged. OD. have small standard deviations. E. have rounding errors.

Answers

a) Option A, A x2 statistic provides strong evidence in favor alternative hypothesis if its value is a large positive number.

b) Option B, The null hypothesis that sex and personal goals are not related vs. the alternative hypothesis that sex and personal goals are related.

c) Option B, The variables considered in a chi-squared test used to evaluate a contingency table B. are categorical.

(a) A x2 statistic provides strong evidence in favor of the alternative hypothesis if its value is a large positive number. The x2 statistic is used in hypothesis testing to determine whether there is a significant difference between observed and expected frequencies. A large positive value indicates that the observed frequencies are significantly different from the expected frequencies, which supports the alternative hypothesis.

(b) The hypotheses being tested in a chi-squared test on a contingency table are the null hypothesis that sex and personal goals are not related vs. the alternative hypothesis that sex and personal goals are related. This test determines whether there is a significant association between two categorical variables.

(c) The variables considered in a chi-squared test used to evaluate a contingency table are categorical. These variables cannot be averaged or assumed to be normally distributed. The chi-squared test is used to analyze the relationship between two or more categorical variables, where each variable has a discrete set of categories.

Learn more about statistics at

https://brainly.com/question/29093686

#SPJ4

Determine whether the set StartSet left bracket Start 3 By 1 Matrix 1st Row 1st Column 1 2nd Row 1st Column 0 3rd Row 1st Column negative 3 EndMatrix right bracket comma left bracket Start 3 By 1 Matrix 1st Row 1st Column negative 3 2nd Row 1st Column 1 3rd Row 1st Column 6 EndMatrix right bracket comma left bracket Start 3 By 1 Matrix 1st Row 1st Column 1 2nd Row 1st Column negative 1 3rd Row 1st Column 0 EndMatrix right bracket EndSet 1 0 −3 , −3 1 6 , 1 −1 0 is a basis for set of real numbers R cubedℝ3. If the set is not a​ basis, determine whether the set is linearly independent and whether the set spans set of real numbers R cubedℝ3. Which of the following describe the​ set?

Answers

The set [1 0 −3 , −3 1 6 , 1 −1 0] is not a basis for set of real numbers R cubed(ℝ3).

The reason why it is not a basis is because it is not linearly independent. However, the set does span set of real numbers R cubed(ℝ3).To determine if the given set is a basis for set of real numbers R cubed(ℝ3), we need to test for linear independence and for span.Linear independence

The given set is said to be linearly independent if and only if the only solution to the equation a[1 0 −3] + b[−3 1 6] + c[1 −1 0] = [0 0 0] is the trivial solution where a, b and c are constants.If the given set is linearly independent then it is a basis for R3; if it is not linearly independent, it is not a basis for R3.

SpanThe given set is said to span R3 if every vector in R3 can be written as a linear combination of vectors in the given set.

If the given set spans R3, then it can be considered a basis for R3.For us to test if the given set is linearly independent, we can form a matrix by placing the three given vectors into the columns of a 3 x 3 matrix as follows:[1 0 1] [−3 1 −1] [−3 6 0]

By expanding the determinant of the matrix above, we get: det(A) = 0 - 0 - (-3) = 3

Since the determinant is non-zero, we can say that the given set is linearly independent. Since the given set is linearly independent, we can then use it to span R3. Hence the given set does not form a basis for R3 but it is linearly independent and spans R3.

To know more about linearly independent click on below link:

https://brainly.com/question/30720942#

#SPJ11

Please help it’s for tmr, I only have 18 minutes left
Leo has a number of toy soldiers between 27 and 54. If he wants to group them four by four, there are none left, seven by seven, 6 remain, five by five, 3 remain. How many toy soldiers are there?
The answer is 48 but I need step by step explanation

Answers

Leo might therefore have 36 or 48 toy soldiers, which is a choice between the two numbers.

What is the greatest number that is possible?

The attempt to demonstrate that your integer is larger than anyone else's integer has persisted through the ages, despite their being more numbers than there are atoms in the universe. The largest number that is frequently used is a googolplex (10googol), which equals 101¹⁰⁰.

We'll name Leo's collection of toy soldiers "x" the amount. We are aware of:

We can infer x to be one of the following figures from the first condition: 28, 32, 36, 40, 44, 48, or 52.

To find out which of these integers meets the other two requirements, we can try each one individually:

x + 6 = 34 and x + 3 = 31, neither of which is a multiple of five, if x = 28.

X + 6 = 38 and X + 3 = 35, none of which is a multiple of 5, follow if x = 32.

When x = 36, x + 6 = 42, a multiple of 7, and x + 3 = 39, a multiple of 5, follow. This might be the answer.

x + 6 = 46 and x + 3 = 43, neither of which is a multiple of five, if x = 40.

x + 6 = 50 and x + 3 = 47, neither of which is a multiple of five, if x = 44.

When x = 48, x + 6 = 54, a multiple of 7, and x + 3 = 51, a multiple of 5, follow.

x + 6 = 58 and x + 3 = 55, neither of which is a multiple of five, if x = 52.

To know more about two numbers visit:-

https://brainly.com/question/1814627

#SPJ1

Complete the table by finding the balance A when P dollars is invested at rate r for t years and compounded n times per year. (Round your answers to the nearest cent. )

P = $1300, r = 8. 5%, t = 11 years

n A

1 $

2 $

4 $

12 $

365 $

Continuous $

Answers

The complete table for the amount balance, A when P dollars is invested at rate r for t years and compounded n times per year is present in above figure 2.

The compound interest formula is written as A = P( 1 + r/n)ⁿᵗ

where, A--> total Amount of money after t years

P --> Principal

r --> Annual rate of interest (as a decimal)

t --> Number of years:

n--> number of times interest is compounded per year

Here, principle, P = $1300, rate of interest, r = 8.5% = 0.085 , time periods, t = 11 years. We have to complete the above table for compound interest.

Case 1: n = 1

Substitute the known values in above formula, A = 1300( 1 + 0.085/1)¹¹

= 1300( 1.085)¹¹

= 3,189.12

Case 2: n = 2

A = 1300( 1 + 0.085/2)²²

= 1300( 2.085/2)²²

= 1300( 1.0425)²²

= 3,248.01

I'll let you work out the cases where n = 4, 12 and 365 since all you need to do is place those in for n as done in the 1st 2 cases. For the Compounded continuously case, the formula becomes,

[tex] A = Pe^{rt}[/tex]

Where: A-> Total amount of money after t years

P --> Principal Amount

e --> Natural log constant:

r = Annual rate of interest (as a decimal)

Case: Continuous: e = 2.71828 (approx), r = 0.085

A = 1300( 2.71828)⁰·⁰⁸⁵⁽¹¹⁾

= 1300(e)⁰·⁹³⁵ = 3,311.34

Hence, required value is $3,311.3775.

For more information about compound interest, visit:

https://brainly.com/question/2455673

#SPJ4

Complete question :

The above table completes the question.

Complete the table by finding the balance A when P dollars is invested at rate r for t years and compounded n times per year. (Round your answers to the nearest cent. ) P = $1300, r = 8. 5%, t

= 11 years

10) Find the vertex form of the parabola.

Answers

Step-by-step explanation:

4x + y^2 + 2y =  - 5

4x = - y^2 -2y - 5          

4x = - (y^2 + 2y)   - 5      'complete the square' for y

4x = - ( y +1)^2   + 1    - 5

 x = - 1/4 ( y+1)^2  - 1             vertex is at -1, -1

-14x+2y^2-8y -20 = 0

14x = 2y^2 -8y-20

14x = 2 ( y^2 - 4y) - 20     complete the square for y

14x = 2(y-2)^2  -8 - 20

x = 1/7 ( y-2)^2 - 2                Vertex is at   -2 , 2

1. Find the given derivative by finding the first few derivatives and observing the pattern that occurs. (d115/dx115(sin(x)). 2. For what values of x does the graph of f have a horizontal tangent? (Use n as your integer variable. Enter your answers as a comma- separated list.) f(x) = x + 2 sin(x).

Answers

The values of x such that the graph of f has a horizontal tangent are;x = 2π/3 + 2πn, 4π/3 + 2πn, where n is an integer.

1. The given derivative can be found by finding the first few derivatives and observing the pattern that occurs as shown below;Differentiating sin x with respect to x gives the derivative cos x. Continuing this process, the pattern that emerges is that sin x changes sign for every odd derivative, and stays the same for every even derivative. Therefore the 115th derivative of sin x can be expressed as follows;(d115/dx115)(sin x) = sin x, for n = 58 (where n is an even number)2. To find the values of x such that the graph of f has a horizontal tangent, we differentiate f with respect to x, and then solve for x such that the derivative equals zero. We have;f(x) = x + 2sin xDifferentiating f(x) with respect to x gives;f'(x) = 1 + 2cos xFor a horizontal tangent, f'(x) = 0, thus;1 + 2cos x = 02cos x = -1cos x = -1/2The solutions of the equation cos x = -1/2 are;x = 2π/3 + 2πn or x = 4π/3 + 2πnwhere n is an integer. Therefore the values of x such that the graph of f has a horizontal tangent are;x = 2π/3 + 2πn, 4π/3 + 2πn, where n is an integer.

Learn more about Graph

brainly.com/question/17267403

#SPJ11

What is the slope of the line described by the equation below?
y = -6x +3
O A. -6
() в. -з
O C. 6
OD. 3
SUBMIT

Answers

This line is in slope intercept form, which is represented by the equation y=Mx+B.

In this form, M represents the slope of a line, and B represents the line’s y-intercept.

In your equation, y=-6x+3, M is -6 and B is 3. So, your y-intercept is 3 and your slope is -6.

Factorise fully - 4x² - 16x

Answers

Answer: 4x(x - 4)

Step-by-step explanation:

4x² - 16x = 4x(x - 4)

Now we can see that the expression inside the parentheses can also be factored:

x - 4 = (x - 4)

So the fully factorized expression is:

4x² - 16x = 4x(x - 4) = 4x(x - 4)

Answer:

- 4x( x + 4 )

Step-by-step explanation:

[tex]\large{\pmb{\sf{ - 4x^{2} - 16x}}}[/tex]

[tex]\large{\underline{\underline{\sf{Taking \: Out \: {\green{4}} \: As \: Common:-}}}}[/tex]

[tex]\large{\pmb{\sf{\leadsto{- 4(x^{2} + 4x)}}}}[/tex]

[tex]\large{\underline{\underline{\sf{Taking \: Out \: {\green{x}} \: As \: Common:-}}}}[/tex]

[tex]\large{\purple{\boxed{\pmb{\sf{\leadsto{- 4x(x + 4)}}}}}}[/tex]

━━━━━━━━━━━━━━━━━━━━━━

[tex]\star \: {\large{\underline{\underline{\pink{\mathfrak{More:-}}}}}} \: \star[/tex]

[tex]\large{\dashrightarrow}[/tex] Two positive always makes positive sign when multiplied.

[tex]\large{\dashrightarrow}[/tex] Two negatives always makes positive sign when multiplied.

[tex]\large{\dashrightarrow}[/tex] A positive and a negative always makes negative sign when multiplied.

[tex]\large{\dashrightarrow}[/tex] The sum of two positives is always positive with a positive sign.

[tex]\large{\dashrightarrow}[/tex] The sum of two negatives is always positive with a negative sign.

[tex]\large{\dashrightarrow}[/tex] The sum of a positive and a negative is always negative with the sign of whose number is greater.

Dentify the solution to the following system of equations




Identify the solution to the following system of equations




No solution

(0. 29, 2. 73)

(0. 78, 0. 05) and (-4. 32, 12. 80)

( -1, 4. 5)

Answers

If m = 3/2, the two equations in the system are dependent and have an infinite number of solutions.

To do this, we can set the coefficients of x and y in the two equations equal to each other:

6 - 4m = 2k(2m - 1)

2n - 7 = 3k

Simplifying these equations, we get:

8m - 6 = 4km

2n - 7 = 3k

We can solve the first equation for k:

k = (8m - 6)/(4m)

Substituting this into the second equation and solving for n, we get:

n = (16m - 29)/(8m - 12)

Now we need to check if there are any values of m for which the equations are proportional. We can do this by checking if the expressions for k and n are the same for all values of m.

We find that the expressions for k and n are the same for m = 3/2.

To know more about equation here

https://brainly.com/question/10413253

#SPJ4

Complete Question:

Determine the values of m and n so that the following system of linear equations have infinite number of solutions:

(2m−1)x+3y−5=0

3x+(n−1)y−2=0

Scientists determined that the cause of death in many prawns off the coast of Chile was a nutrient deficiency. So, they set out to determine if the distribution of plants in the ocean near the coast was out of proportion when compared to the ideal environment: 40% Kelp, 25% Phytoplankton, 25% Coral and 10% Other (mostly nutrient-low seaweed). In randomly chosen areas along the coast, they sampled 240 plants.
KELP PHYTOPLANKTON CORAL OTHER
84 67 57 32
In an ideal environment how many of the 240 plants would you expect to be Kelp?
If a goodness of fit test is conducted, what is the null Hypothesis?
If a goodness of fit test is conducted, what is the alternative Hypothesis?
What is the probability of getting the observed values or values as extreme from the ideal?
Is there enough evidence to conclude that the environment for prawns is not ideal? Base this conclusion on p-value and a level of significance of 0.05 or 5%.

Answers

Answer:

Step-by-step explanation:

stuck on this question need some help

Answers

Answer:

1. The graphs of f(x) and h(x) are both quadratic functions with a minimum point. However, the minimum point of f(x) is located at (6,0), while the minimum point of h(x) is located at (2,3).

2. The graphs of g(x) and h(x) both open upwards and are quadratic functions. However, the vertex of g(x) is located at the origin (0,0), while the vertex of h(x) is located at (2,3).

3. The graph of g(x) is a simple parabola that opens upwards, while the graphs of f(x) and h(x) are more complex parabolas with a minimum point and an upward opening. The graph of f(x) is centered at (6,0), while the graph of h(x) is centered at (2,3).

Why is pi important in math?

Answers

Pi is used to find the area and circumference of a circle

if x < y < z and all three are consecutive non-zero integers, then which of the following must be a positive odd integer?

Answers

Option (A) x+1 is a positive odd integer.

Given that, x < y < z and all three are consecutive non-zero integers.Let the first number be x, then the other two consecutive non-zero integers will be (x+1) and (x+2).To find out the positive odd integer among these, let us take each of them and verify if they are positive odd integers.∴ x+1 is odd, x+2 is even∴ x+1 is the only positive odd integer out of the three.

Learn more about Non-zero integers

brainly.com/question/2032387

#SPJ11

Either use an appropriate theorem to show that the given set, W, is a vector space, or find a specific example to the contrary.W = {[\begin{array}{ccc}a\\b\\c\\\d\end{array}\right] : 3a+b=c, a+b+2c=2d}

Answers

An appropriate theorem to show that the given set, W, is a vector space. A specific example can be

[tex]\left[\begin{array}{ccc}p\\q\\r\end{array}\right][/tex] , -p- -3q = s  and 3p = -2s - 3r

             

Sets represent values ​​that are not solutions. B. The set of all solutions of a system of homogeneous equations OC.

The set of solutions of a homogeneous equation. Thus the set W = Null A. The null space of n homogeneous linear equations in the mx n matrix A is a subspace of Rn. Equivalently, the set of all solutions of the unknown system Ax = 0 is a subspace of R.A.

The proof is complete because W is a subspace of R2. The given set W must be a vector space, since the subspaces are themselves vector spaces. B. The proof is complete because W is a subspace of R. The given set W must be a vector space, since the subspaces are themselves vector spaces.

The proof is complete because W is a subspace of R4. The given set W must be a vector space, since the subspaces are themselves vector spaces. outside diameter. The proof is complete because W is a subspace of R3. The given set W must be a vector space, since the subspaces are themselves vector spaces.

Let W be the set of all vectors of the right form, where a and b denote all real numbers. Give an example or explain why W is not a vector space. 8a + 3b -4 8a-7b. Select the correct option below and, if necessary, fill in the answer boxes to complete your selection OA. The set pressure is

S = {(comma separated vectors as required OB. W is not a vector space because zero vectors in W and scalar sums and multiples of most vectors are not in W because their second (intermediate) value is not equal to -4. OC. W is not a vector space because not all vectors U, V and win W have the properties

u +v =y+ u and (u + v)+w=u + (v +W).

Learn more about Vector Space:

https://brainly.com/question/16205930

#SPJ4

a normal distribution of exam scores has a standard deviation of 8. a score that is 12 points above the mean would have a z-score of: a score that is 20 points below the mean would have a z-score of:

Answers

The standard deviation of a normal distribution of exam scores is 8. A score that is 12 points above the mean would have a z-score of 1.5, and a score that is 20 points below the mean would have a z-score of -2.5.

What is the z-score?

The z-score can be calculated by dividing the difference between a data value and the mean of the data set by the standard deviation of the data set.

The z-score of a score that is 12 points above the mean in a normal distribution of exam scores with a standard deviation of 8.

z = (x−μ)/σ = (x−μ)/σ = (12−0)/8 = 1.5

The z-score of a score that is 12 points above the mean in a normal distribution of exam scores with a standard deviation of 8 is 1.5.

The z-score of a score that is 20 points below the mean in a normal distribution of exam scores with a standard deviation of 8.

z = {x-μ}/{σ}  = {-20-0}/{8}  = −2.5

The z-score of a score that is 20 points below the mean in a normal distribution of exam scores with a standard deviation of 8 is -2.5.

Learn more about Z-score here:

https://brainly.com/question/15016913

#SPJ11

Other Questions
Which of the following may the president do to control the implementation of a policy agenda?A The president can veto particular items or language in a bill he disagrees with while enacting the remainder of the billB The president can veto a congressional bill that has passed the House and the SenateC The president can refuse to spend money that Congress appropriatedD The president can impeach selected members of CongressWhich of the following may the president do to control the implementation of a policy agenda?A The president can veto particular items or language in a bill he disagrees with while enacting the remainder of the billB The president can veto a congressional bill that has passed the House and the SenateC The president can refuse to spend money that Congress appropriatedD The president can impeach selected members of Congress The Kingston County Basketball Conference (KCBC) is an amateur basketball association. Each city in the county has one team that represents it. Each team has a maximum of twelve players and a minimum of nine players. Each team also has up to three coaches (offensive, defensive, and PT coaches.) Each team plays two games (home and visitor) against each of the other teams during the season.a. Draw the ER diagram.b. Create a relational model based on the ER diagram. your ______________ is the unique combination of feelings, thoughts, and behavior that makes you different from everyone. Mr. Miles, a 65-year-old man, is visiting his health care pro-vider for an annual checkup. He tells you that he has beenexperiencing difficulty with urination, including increased fre-quency, urinating more at night, difficulty starting his urinestream, and dribbling after urination. He states that after urina-tion, he feels like his bladder is not emptied.1. What diagnosis is suggested by Mr. Miles's symptoms?2. What classes of drugs do you expect will be prescribedfor Mr. Miles, and how will these drugs work?3. What important teaching points will you include regardingthe diagnosis and its treatment? Un smbolo es _________ de una idea. La semilla la representacin el argumento el opuesto 2. La lupa en esta obra representa _________. La guerra la ceguera la opresin la creacin artstica 3. Las aves en la obra representan _________. La libertad la muerte el miedo la sabidura 4. El personaje de la obra parece _________ mundo exterior. Interesado en el inspirado por el aislado del creado por el 5. La mquina de pinturas puede reflejar la influencia de _________. Los artistas de la vanguardia la guerra Benjamn Peret su padre During the Meiji Restoration, fought to as fast as possible and as a result changed its significantly. fill in the blank. by threatening trade sanctions, one country can convince another country to open its markets. The generally accepted belief in the United States that women are more nurturing and better able to care for children is an example of? mrs. grundy believes that boarding schools provide the best education because after attending such a school, she found herself well prepared for college. the fallacy in her thinking is If 110 grams of potassium chloride are mixed with 100 grams of water at 20C, how much will not dissolve? effective texting during workplace meetings includes which of the following? (select all that apply.) 1. List and briefly explain the rules that should apply to the use of elevators when responding to a fire incident in a high-rise building.2.In your own words, explain Stack Effect as it relates to high-rise buildings. 3. You have responded to a fire incident in a high-rise building. When you arrive the Incident Commander tasks you with the assignment of Lobby Control. What are your duties? please select the two correct answers to test your understanding of the primary goals of antimicrobial treatment. identify the letter that indicates the organ that directs sperm towards the seminal vesicles during ejavulation Identify the option that defines how positions are measured on the background in the following background property:background: color url(url) position / size repeat attachmentorigin clip; Classify each titration curve as representing a strong acid titrated with a strong base, a strong base titrated with a strong acid, a weak acid titrated with a strong base, a weak basetaed with a strong acid, or a polyprotic acid titrated with a strong base. Strong acid/Strong base/ strong base Weak acid strong base Weak base Polyprotic acid strong acid strong acid strong base mL of titrant mL of titrant mL of titrant mL of titrant mL of titrant web sites provide access to specic types of useful information, such as maps and driving directions. a.educational b.reference c.product information d.news Businesses typically have many systems, some internally developed, some purchased, and others acquired. Businesses want these systems to communicate with one another and provide users with easier access to data. There are three systems configurations that consolidate and coordinate data across multiple locations. These three include centralized systems, decentralized systems, and distributed systems. Each configuration has advantages and disadvantages.Which of the following is a disadvantage of decentralized systems?-It is the most expensive option as it involves higher maintenance costs.-Since each location has its own system, there is an increase in security risk as more systems must be protected and monitored.- The risk of business disruption is greater because any disruption impacts the entire system.-It is more difficult to implement as it is the most complex of the three. what does president coolidges perspective on capitalism and the role of the president demonstrate about republican party policy in the 1920s and its relationship with the progressivism of the early twentieth century? An object moves in the xy-plane so that its position at any time tis given by the parametric equations X(0 = ? _ 3/2+2andy (t) = Vt? + 16.What is the rate of change of ywith respect t0 when t = 3 1/90 1/15 3/5 5/2'